The only function that forms a homomorphism from $mathbb Z_n$ to $mathbb Z_n$












0












$begingroup$


Define $M_a : Z_n → Z_n$ by $M_a([x]) = [ax]$



Prove that



(a) $M_a$ : $(Z_n, +) → (Z_n, +)$ is a homomorphism.



(b) Let φ : $(Z_n, +) → (Z_n, +)$ be a homomorphism. Prove that φ = $M_a$ for some a ∈ Z.



(c) For two homomorphism φ, ψ : $(Z_n, +) → (Z_n, +)$, prove that if φ = $M_a and ψ = M_b$, then for
their composition φ ◦ ψ = $M_{ab}$.



My attempt:
(a) is just using the definition of homomorphism and knowing that $[ax+ay]_n=[ax]_n +[ay]_n$



(c) involved simply using definitions of these functions as well.



I do not know how to prove (b):
Using the fact that the function has been given to be a homomorphism, what I do is:



$ψ(0 + a) = ψ(0) + ψ (a)$



This means $ψ (a) = [0]_n$
But I do not see how this will help me conclude the function must be of the form $M_a$
Could someone please help or provide an alternative?










share|cite|improve this question











$endgroup$








  • 1




    $begingroup$
    Note that $varphi([k]) = varphi([1]+cdots+[1]) = varphi([1])+cdots+varphi([1])$, where the sums have $k$ summands.
    $endgroup$
    – Christoph
    Dec 12 '18 at 17:30
















0












$begingroup$


Define $M_a : Z_n → Z_n$ by $M_a([x]) = [ax]$



Prove that



(a) $M_a$ : $(Z_n, +) → (Z_n, +)$ is a homomorphism.



(b) Let φ : $(Z_n, +) → (Z_n, +)$ be a homomorphism. Prove that φ = $M_a$ for some a ∈ Z.



(c) For two homomorphism φ, ψ : $(Z_n, +) → (Z_n, +)$, prove that if φ = $M_a and ψ = M_b$, then for
their composition φ ◦ ψ = $M_{ab}$.



My attempt:
(a) is just using the definition of homomorphism and knowing that $[ax+ay]_n=[ax]_n +[ay]_n$



(c) involved simply using definitions of these functions as well.



I do not know how to prove (b):
Using the fact that the function has been given to be a homomorphism, what I do is:



$ψ(0 + a) = ψ(0) + ψ (a)$



This means $ψ (a) = [0]_n$
But I do not see how this will help me conclude the function must be of the form $M_a$
Could someone please help or provide an alternative?










share|cite|improve this question











$endgroup$








  • 1




    $begingroup$
    Note that $varphi([k]) = varphi([1]+cdots+[1]) = varphi([1])+cdots+varphi([1])$, where the sums have $k$ summands.
    $endgroup$
    – Christoph
    Dec 12 '18 at 17:30














0












0








0





$begingroup$


Define $M_a : Z_n → Z_n$ by $M_a([x]) = [ax]$



Prove that



(a) $M_a$ : $(Z_n, +) → (Z_n, +)$ is a homomorphism.



(b) Let φ : $(Z_n, +) → (Z_n, +)$ be a homomorphism. Prove that φ = $M_a$ for some a ∈ Z.



(c) For two homomorphism φ, ψ : $(Z_n, +) → (Z_n, +)$, prove that if φ = $M_a and ψ = M_b$, then for
their composition φ ◦ ψ = $M_{ab}$.



My attempt:
(a) is just using the definition of homomorphism and knowing that $[ax+ay]_n=[ax]_n +[ay]_n$



(c) involved simply using definitions of these functions as well.



I do not know how to prove (b):
Using the fact that the function has been given to be a homomorphism, what I do is:



$ψ(0 + a) = ψ(0) + ψ (a)$



This means $ψ (a) = [0]_n$
But I do not see how this will help me conclude the function must be of the form $M_a$
Could someone please help or provide an alternative?










share|cite|improve this question











$endgroup$




Define $M_a : Z_n → Z_n$ by $M_a([x]) = [ax]$



Prove that



(a) $M_a$ : $(Z_n, +) → (Z_n, +)$ is a homomorphism.



(b) Let φ : $(Z_n, +) → (Z_n, +)$ be a homomorphism. Prove that φ = $M_a$ for some a ∈ Z.



(c) For two homomorphism φ, ψ : $(Z_n, +) → (Z_n, +)$, prove that if φ = $M_a and ψ = M_b$, then for
their composition φ ◦ ψ = $M_{ab}$.



My attempt:
(a) is just using the definition of homomorphism and knowing that $[ax+ay]_n=[ax]_n +[ay]_n$



(c) involved simply using definitions of these functions as well.



I do not know how to prove (b):
Using the fact that the function has been given to be a homomorphism, what I do is:



$ψ(0 + a) = ψ(0) + ψ (a)$



This means $ψ (a) = [0]_n$
But I do not see how this will help me conclude the function must be of the form $M_a$
Could someone please help or provide an alternative?







group-theory






share|cite|improve this question















share|cite|improve this question













share|cite|improve this question




share|cite|improve this question








edited Dec 12 '18 at 17:48









user43210

406




406










asked Dec 12 '18 at 17:21









childishsadbinochildishsadbino

1148




1148








  • 1




    $begingroup$
    Note that $varphi([k]) = varphi([1]+cdots+[1]) = varphi([1])+cdots+varphi([1])$, where the sums have $k$ summands.
    $endgroup$
    – Christoph
    Dec 12 '18 at 17:30














  • 1




    $begingroup$
    Note that $varphi([k]) = varphi([1]+cdots+[1]) = varphi([1])+cdots+varphi([1])$, where the sums have $k$ summands.
    $endgroup$
    – Christoph
    Dec 12 '18 at 17:30








1




1




$begingroup$
Note that $varphi([k]) = varphi([1]+cdots+[1]) = varphi([1])+cdots+varphi([1])$, where the sums have $k$ summands.
$endgroup$
– Christoph
Dec 12 '18 at 17:30




$begingroup$
Note that $varphi([k]) = varphi([1]+cdots+[1]) = varphi([1])+cdots+varphi([1])$, where the sums have $k$ summands.
$endgroup$
– Christoph
Dec 12 '18 at 17:30










1 Answer
1






active

oldest

votes


















3












$begingroup$

Let $ainmathbb Z$ be such that $varphi(1)=[a]$. Then $varphi(1)=M_a(1)$. Since $mathbb{Z}_n=langle1rangle$, you deduce from this that $varphi=M_a$.






share|cite|improve this answer









$endgroup$













    Your Answer





    StackExchange.ifUsing("editor", function () {
    return StackExchange.using("mathjaxEditing", function () {
    StackExchange.MarkdownEditor.creationCallbacks.add(function (editor, postfix) {
    StackExchange.mathjaxEditing.prepareWmdForMathJax(editor, postfix, [["$", "$"], ["\\(","\\)"]]);
    });
    });
    }, "mathjax-editing");

    StackExchange.ready(function() {
    var channelOptions = {
    tags: "".split(" "),
    id: "69"
    };
    initTagRenderer("".split(" "), "".split(" "), channelOptions);

    StackExchange.using("externalEditor", function() {
    // Have to fire editor after snippets, if snippets enabled
    if (StackExchange.settings.snippets.snippetsEnabled) {
    StackExchange.using("snippets", function() {
    createEditor();
    });
    }
    else {
    createEditor();
    }
    });

    function createEditor() {
    StackExchange.prepareEditor({
    heartbeatType: 'answer',
    autoActivateHeartbeat: false,
    convertImagesToLinks: true,
    noModals: true,
    showLowRepImageUploadWarning: true,
    reputationToPostImages: 10,
    bindNavPrevention: true,
    postfix: "",
    imageUploader: {
    brandingHtml: "Powered by u003ca class="icon-imgur-white" href="https://imgur.com/"u003eu003c/au003e",
    contentPolicyHtml: "User contributions licensed under u003ca href="https://creativecommons.org/licenses/by-sa/3.0/"u003ecc by-sa 3.0 with attribution requiredu003c/au003e u003ca href="https://stackoverflow.com/legal/content-policy"u003e(content policy)u003c/au003e",
    allowUrls: true
    },
    noCode: true, onDemand: true,
    discardSelector: ".discard-answer"
    ,immediatelyShowMarkdownHelp:true
    });


    }
    });














    draft saved

    draft discarded


















    StackExchange.ready(
    function () {
    StackExchange.openid.initPostLogin('.new-post-login', 'https%3a%2f%2fmath.stackexchange.com%2fquestions%2f3036975%2fthe-only-function-that-forms-a-homomorphism-from-mathbb-z-n-to-mathbb-z-n%23new-answer', 'question_page');
    }
    );

    Post as a guest















    Required, but never shown

























    1 Answer
    1






    active

    oldest

    votes








    1 Answer
    1






    active

    oldest

    votes









    active

    oldest

    votes






    active

    oldest

    votes









    3












    $begingroup$

    Let $ainmathbb Z$ be such that $varphi(1)=[a]$. Then $varphi(1)=M_a(1)$. Since $mathbb{Z}_n=langle1rangle$, you deduce from this that $varphi=M_a$.






    share|cite|improve this answer









    $endgroup$


















      3












      $begingroup$

      Let $ainmathbb Z$ be such that $varphi(1)=[a]$. Then $varphi(1)=M_a(1)$. Since $mathbb{Z}_n=langle1rangle$, you deduce from this that $varphi=M_a$.






      share|cite|improve this answer









      $endgroup$
















        3












        3








        3





        $begingroup$

        Let $ainmathbb Z$ be such that $varphi(1)=[a]$. Then $varphi(1)=M_a(1)$. Since $mathbb{Z}_n=langle1rangle$, you deduce from this that $varphi=M_a$.






        share|cite|improve this answer









        $endgroup$



        Let $ainmathbb Z$ be such that $varphi(1)=[a]$. Then $varphi(1)=M_a(1)$. Since $mathbb{Z}_n=langle1rangle$, you deduce from this that $varphi=M_a$.







        share|cite|improve this answer












        share|cite|improve this answer



        share|cite|improve this answer










        answered Dec 12 '18 at 17:25









        José Carlos SantosJosé Carlos Santos

        163k22131234




        163k22131234






























            draft saved

            draft discarded




















































            Thanks for contributing an answer to Mathematics Stack Exchange!


            • Please be sure to answer the question. Provide details and share your research!

            But avoid



            • Asking for help, clarification, or responding to other answers.

            • Making statements based on opinion; back them up with references or personal experience.


            Use MathJax to format equations. MathJax reference.


            To learn more, see our tips on writing great answers.




            draft saved


            draft discarded














            StackExchange.ready(
            function () {
            StackExchange.openid.initPostLogin('.new-post-login', 'https%3a%2f%2fmath.stackexchange.com%2fquestions%2f3036975%2fthe-only-function-that-forms-a-homomorphism-from-mathbb-z-n-to-mathbb-z-n%23new-answer', 'question_page');
            }
            );

            Post as a guest















            Required, but never shown





















































            Required, but never shown














            Required, but never shown












            Required, but never shown







            Required, but never shown

































            Required, but never shown














            Required, but never shown












            Required, but never shown







            Required, but never shown







            Popular posts from this blog

            Bundesstraße 106

            Le Mesnil-Réaume

            Ida-Boy-Ed-Garten